2023-09-20 09:49:44 -07:00

146 lines
2.8 KiB
TeX

\section{Defining $e$}
\problem{}
Recall and prove the binomial theorem.
\begin{solution}
The binomial theorem:
$$(a + b)^n = \sum_{i=0}^{n} \binom{n}{i}a^ib^{n-i}$$
We usually prove this by induction.
\end{solution}
\vfill
\problem{}<e_n>
Prove the following:
$$ \bigg(1 + \frac{1}{n} \bigg)^n < 3 - \frac{1}{n}\ \text{for all } n = 3, 4, 5, ...$$
This proves that that the sequence $e_n = (1 + \frac{1}{n})^n$, $n = 1, 2, ...$ is bounded from above, \par
since $e_n < 3\ \forall n \in \mathbb{N}$.
\begin{solution}
$$
\bigg(1 + \frac{1}{n}\bigg)^n =
\sum_{k=0}^n \binom{n}{k}\frac{1}{n^i} =
2 + \sum_{k=2}^n \binom{n}{k}\frac{1}{n^i} =
2 + \sum_{k=2}^n \frac{1}{k!}\frac{(n)(n-1)...(n-k+1)}{n^k} <
2 + \sum_{k=2}^n \frac{1}{k!}
$$
$$
2 + \sum_{k=2}^n \frac{1}{k!} <
2 + \sum_{k=2}^n \frac{1}{k(k-1)} =
2 + 1 - \frac{1}{n} = 3 - \frac{1}{n}
$$
\end{solution}
\vfill
\pagebreak
\theorem{Bernoulli's inequality}<bernoulli>
$(x + 1)^n \geq 1 + nx$ for $x \geq -1$ and $n \in \mathbb{N}$
\problem{}
Use induction to prove \ref{bernoulli}
\vfill
\problem{}<e_n_inc>
Use \ref{bernoulli} to prove that the $e_n$ defined in \ref{e_n} is monotonically increasing.
\begin{solution}
We want to show that the following is true:
$$
\bigg(
1 + \frac{1}{n + 1}
\bigg)^{n+1}
>
\bigg(
1 + \frac{1}{n}
\bigg)^n
$$
This inequality is equivalent to
$$
\Bigg(
\frac{
1 + \frac{1}{n+1}
}{
1 + \frac{1}{n}
}
\Bigg)^{n+1}
>
\bigg(
1 + \frac{1}{n}
\bigg)^{-1}
= \frac{1}{1 + \frac{1}{n}}
= \frac{n}{n+1}
= 1 - \frac{1}{n+1}
$$
Also,
$$
\frac{
1 + \frac{1}{n+1}
} {
1 + \frac{1}{n}
}
= 1 - \frac{1}{(n + 1)^2}
$$
\ref{bernoulli} tells us that
$$
\bigg(
1 - \frac{1}{(n+1)^2}
\bigg) ^ {n+1}
= 1 - \frac{n+1}{(n+1)^2}
= 1 - \frac{1}{n+1}
$$
Since this is equivalent to our original inequality, we are done.
\end{solution}
\vfill
\pagebreak
\definition{}
\ref{e_n}, \ref{e_n_inc}, and \ref{limexists} tell us that $e_n$ has a limit. \par
Let us define $e$:
$$e = \lim_{n\to\infty}{e_n} = \lim_{n\to\infty}{\bigg(1 + \frac{1}{n} \bigg)^n}$$
\vfill
\problem{}
Derive the formula for $V(t)$ if the annual rate $r$ is compounded continuously.
\begin{solution}
For a rate $r$ compounded $n$ times per year, we have $V(t) = P(1 + \frac{r}{n})^{nt}$.
$\lim_{n\to\infty}{(V(t))} = \lim_{n\to\infty}{P(1 + \frac{r}{n})^{nt}}$
Let $a = \frac{n}{r}$ \par
$\lim_{n\to\infty}{a} = \infty$, so $\lim_{n\to\infty}{V(t)} = \lim_{a\to\infty}{V(t)}$ \par
Substituting $a$ for $n$, we get \par
$P(1 + \frac{r}{n})^{nt} = P(1 + \frac{1}{a})^{art}$ \par
And finally, we can evaluate \par
$\lim_{a\to\infty}{P (1 + \frac{1}{a})^{art}} = Pe^{rt}$
\end{solution}
\vfill
\pagebreak